What is the total value of digit 7 in the number 32.8794​

Answers

Answer 1

Answer:

hundredth

Step-by-step explanation:

numbers after the decimal point start counting from

tenth

hundredth

Answer 2

Answer:

0.07 or 7 hundredths

Step-by-step explanation:

(12.3456)- I'll use this as my example

1-tens place

2-ones place

3- tenths place

4= hundredths place

5-thousandths place

6- im pretty sure its the 10 thousandths place


Related Questions

No links, will mark brainliest!!
SA is the perpendicular bisector of TR . m A) 45
B) 60
C) 39
D) 180

Answers

Answer:

if you are supposed to find m<STR, then the answer is B

Step-by-step explanation:

Answer:

i thinkits c

Step-by-step explanation:

based on the info given

Si "m" es el lado mayor de un triángulo, dos cuyos ángulos interiores miden 30° y 60°, entonces el perímetro del triángulo es

Answers

Answer:

[tex]perimetro = \dfrac{(3 + \sqrt{3})m}{2}[/tex]

Step-by-step explanation:

triángulo 30°-60°-90°

[tex] 1~:~\sqrt{3}~:~2 [/tex]

[tex] 1m~:~m\sqrt{3}~:~2m [/tex]

[tex] \dfrac{m}{2}~:~\dfrac{m\sqrt{3}}{2}~:~m [/tex]

[tex]perimetro = \dfrac{m}{2} + \dfrac{m\sqrt{3}}{2} + m[/tex]

[tex]perimetro = \dfrac{m}{2} + \dfrac{m\sqrt{3}}{2} + \dfrac{2m}{2}[/tex]

[tex]perimetro = \dfrac{m + m\sqrt{3} + 2m}{2}[/tex]

[tex]perimetro = \dfrac{3m + m\sqrt{3}}{2}[/tex]

[tex]perimetro = \dfrac{(3 + \sqrt{3})m}{2}[/tex]

In the past year Maya watched 34 movies that she thought were very good. She watched 40 movies over the whole year. Of the movies she watched, what percentage did she rate as very good?

Answers

Answer:

90 percent

Step-by-step explanation:

the fraction is 90/100 so the percentage is 90%

Sal is trying to determine which cell phone and service plan to buy for his mother. The first phone costs $100 and $55 per month for unlimited usage. The second phone costs $150 and $51 per month for unlimited usage. How many months will it take for the second phone to be less expensive than the first phone?

The inequality that will determine the number of months, x, that are required for the second phone to be less expensive is
.

The solution to the inequality is
.

Sal’s mother would have to keep the second cell phone plan for at least
months in order for it to be less expensive.

Answers

Answer:

It will take 13 months for the second phone to be less expensive than the first phone.

Step-by-step explanation:

Since Sal is trying to determine which cell phone and service plan to buy for his mother, and the first phone costs $ 100 and $ 55 per month for unlimited usage, while the second phone costs $ 150 and $ 51 per month for unlimited usage, to determine how many months will it take for the second phone to be less expensive than the first phone, the following calculation must be performed:

100 + (55 x 10) = 100 + 550 = 650

150 + (51 x 10) = 150 + 510 = 660

100 + (55 x 12) = 100 + 660 = 760

150 + (51 x 12) = 150 + 612 = 762

100 + (55 x 13) = 100 + 715 = 815

100 + (51 x 13) = 150 + 663 = 813

Therefore, it will take 13 months for the second phone to be less expensive than the first phone.

Answer:

1) 100 + 55x > 150 + 51x

2) x > 12.5

3) 13

Step-by-step explanation:

Got it right on Edge :D

A rectangular tank 60 cm long and 50 cm wide is /5 full of water. When 24 liters of water are added, the water level rises to the brim of the tank. Find the height of the tank. (1 liter is 1000cm3 )

Answers

Answer:

The tank is 10cm high

Step-by-step explanation:

Given

[tex]L=60cm[/tex] -- length

[tex]W=60cm[/tex] -- width

[tex]x = \frac{1}{5}[/tex] --- water lever

[tex]Addition = 24L[/tex]

Required

The height of the tank

Let y represents the remaining fraction before water is added.

So:

[tex]y + x = 1[/tex]

Make y the subject

[tex]y = 1 - x[/tex]

[tex]y = 1 - \frac{1}{5}[/tex]

Solve

[tex]y = \frac{5 - 1}{5}[/tex]

[tex]y = \frac{4}{5}[/tex]

Represent the volume of the tank with v

So:

[tex]y * v = 24L[/tex]

Make v the subject

[tex]v = \frac{24L}{y}[/tex]

Substitute: [tex]y = \frac{4}{5}[/tex]

[tex]v = \frac{24L}{4/5}[/tex]

[tex]v = 30L[/tex]

Represent the height of the tank with h;

So, the volume of the tank is:

[tex]v = lwh[/tex]

Make h the subject

[tex]h = \frac{v}{lw}[/tex]

Substitute values for v, l and w

[tex]h = \frac{30L}{60cm * 50cm}[/tex]

Convert 30L to cm^3

[tex]h = \frac{30*1000cm^3}{60cm * 50cm}[/tex]

[tex]h = \frac{30000cm^3}{3000cm^2}[/tex]

[tex]h = 10cm[/tex]

i need help asap. rn ​

Answers

Answer:

4

Step-by-step explanation:

For this, you need to find the scale factor of two sides that are already given to you.

So, we will have to use the hypotenuse and one of the legs to make sure there is an accurate scale factor.

Hypotenuse:

15 / 5 = 3

Leg:

6 / 2 = 3

____________

So the scale factor is 3.

Using the leg (with the x) we need to divide 12 by the scale factor (3) to give us what x is equal to.

12 / 3 = 4

So, the answer is 4.

2.



What is the difference between the lower quartile rainfall of Florida and the lower quartile rainfall of Pennsylvania?

Answers

Answer:

0.5 inches

Step-by-step explanation:

The lower quartile is the value on the left side of the box plot

lower quartile for Florida = 3.5 inches

lower quartile for Pennsylvania = 3 inches

difference = 3.5 - 3 = 0.5 inches

which algebraic expressions are polynomials​

Answers

Answer:

last option

Step-by-step explanation:

3.9x3 -4.1x2+7.3

g(x)=√8x​
What is the domain of g?

Answers

G=undefined
X:7.74870453
G:7.87334975

increase 30 by 1/2
help please its urgent​

Answers

Answer:

30.5 for decimal form and 30 1 /2 if mixed fraction needed

Helpppp someone please

Answers

Given:

The graph of system of inequalities.

To find:

The system of inequalities.

Solution:

From the given graph it is clear that the shaded region is lies below the line y=10 and the boundary line y=10 is a solid line. So, the sign of inequality must be [tex]\leq[/tex].

[tex]y\leq 10[/tex].

The shaded region lies on the right side of the y-axis. So, [tex]x\geq 0[/tex].

Another boundary line passes through the point (0,1) and (1,2). So, the equation of the line is:

[tex]y-y_1=\dfrac{y_2-y_1}{x_2-x_1}(x-x_1)[/tex]

[tex]y-1=\dfrac{2-1}{1-0}(x-0)[/tex]

[tex]y=1(x)+1[/tex]

[tex]y=x+1[/tex]

The boundary line [tex]y=x+1[/tex] is a dotted line and the shaded region lies above the boundary line, so the sign of inequality must be [tex]>[/tex].

[tex]y> x+1[/tex]

So, the required system of inequalities is:

[tex]y> x+1[/tex]

[tex]y\leq 10[/tex]

[tex]x\geq 0[/tex]

Therefore, the correct option is B.

Which is a simplified form of the expression -5(y – 2) – 7y?
A. -12y + 10
B. 2y - 10
C. -2y + 10
D. 12y - 10

Answers

Answer:

the answer is B

Step-by-step explanation:

-5(y-2)-7y

-5y-10-7y

+7y +7y

answer: 2y-10

Answer:

A. (-12y+10)

Step-by-step explanation:

-5(y-2) - 7y can be simplified to:

-5y + 10 - 7y when you factor the parenthesis.

If you add the y's together, the answer should be:

-12y + 10.

.

Kenneth ran a marathon (26.2) miles) in 5.5 hours. What was Kenneth’s average speed? (Round your answer to the nearest tenth.)

Answers

Answer:

4.8 miles per hour.

Step-by-step explanation:

Average speed=Total distance/Time taken

=26.2miles/5.5hour

=4.8 miles per hour

Use slope to determine if lines AB and CD are parallel, perpendicular, or neither. A(0,2), B(5,4), C(1,8), and D(3,3). What is the slope of line CD? Enter as a ratio in its lowest terms.

Answers

Answer:

Perpendicular, CD has a slope of -5/2

Step-by-step explanation:

Parallel lines have the same slope, perpendicular have slopes that are negative reciprocals, or in other words if one slope is m then the other's slope is -1/m and if it is neither fo those then it is neither parallel or perpendicular.  Also,  if two lines have the same slope you have to make sure they are not the same line.  

Now, to find the slope  when you have two points you use the formula (y2 - y1)/(x2 - x1), where the points are (x1, y1) and (x2, y2).  You can make either point point 1 or point 2.  Just make sure x1 and y1 are from one point and x2 and y2 are from the other point.  

So the two lines are AB and CD.  Since the question asks I will start with CD.  The points are (1,8) and (3,3)  So I will call C point 1 and D point 2.  

(y2 - y1)/(x2 - x1)

(3 - 8)/(3 - 1)

-5/2

Thankfully this is lowest terms.

Now for AB.  A will be point 1 and B will be point 2

(y2 - y1)/(x2 - x1)

(4-2)/(5-0)

2/5

So they are not the same, so what if you take negative 1 over one of the slopes.  

well. -1/(2/5) = -5/2

So these are perpendicular.  

If you would like an explanation for how to find the full equation of a line I can explain that.  

Determine the axis of symmetry of the parabola with x-intercepts at (4, 0) and (12, 0)

Answers

Step-by-step explanation:

hope it helps thank you❤

What is the range of the function f(x) = 4x + 9, given the domain D = {-4, -2, 0, 2}?

Answers

Answer:

Range D = {-7, 1, 9, 17}

Step-by-step explanation:

f(x) = 4x + 9

4(-4) + 9 = -7

4(-2) + 9 = 1

4(0) + 9 = 9

4(2) + 9 = 17

What is the relationship between an angle of elevation and an angle of depression in a right triangle?

Answers

Answer: Angles of Elevation and Depression are used in measuring heights and distances in trigonometric applications using right triangles. These angles are made when we look up or down to view objects. Devices are available to measure angles of elevation and depression. These measured angles can be used in measuring heights and distance which are either tedious or impractical to measure, by modelling the situation into right triangles

Answer:

Angles of Elevation and Depression are used in measuring heights and distances in trigonometric applications using right triangles. These angles are made when we look up or down to view objects. Devices are available to measure angles of elevation and depression. These measured angles can be used in measuring heights and distance which are either tedious or impractical to measure, by modelling the situation into right triangles

Step-by-step explanation:

correct on EDGE2021

In the adjoining parallelogram ABCD, A is joined to any point E on BC. AE and DC produced meet at E Prove that. area of ∆BEF = area of ∆ CDE​

Answers

Answer:

In parallelogram ABCD any point E is taken on the side BC . AE and DC when produced to meet at a point M . Prove that ar(ADM) = ar(ABMC) .

wow vvvvvvvvvvvvvvbbbbbb

I just need to know how I would be able to find x

Answers

Answer:

[tex]x=15[/tex]°

Step-by-step explanation:

The sum of degree measures in a full angle (a circle) is (360) degrees. This means that the sum of all of the angles in this diagram is (360) degrees, as the angles form a full arc. Therefore, one can form an equation by adding up all of the angles and setting the equation equal to (360) degrees. Then one can substitute each angle value with the equation that is used to represent it, simplify, and use inverse operations to solve for the value of (x).

[tex](m<AMB)+(m<BMC)+(m<CMD)+(m<AMD)=(360)[/tex]

Substitute,

[tex](46)+(4x-2)+(9x+6)+(8x-5)=360[/tex]

Simplify,

[tex](46)+(4x-2)+(9x+6)+(8x-5)=360[/tex]

[tex]21x+45=360[/tex]

Inverse operations,

[tex]21x+45=360[/tex]

[tex]21x=315[/tex]

[tex]x=15[/tex]

Solve for x.

Help me please

Answers

Answer:

x = 24

Step-by-step explanation:

Mathematically, in a cyclic quadrilateral; two opposite angles are supplementary

what this mean is that the opposite angles add up to equal 180 degrees

mathematically, we have this as;

(4x + 9) + (3x + 3) = 180

4x + 3x + 3 + 9 = 180

7x + 12 = 180

7x = 180-12

7x = 168

x = 168/7

x = 24

5 positive integers are arranged in ascending order, as follows:
1,9, 9, 10, X
The mean and the median are equal.
Find X.

Answers

Answer:

x = 16

Step-by-step explanation:

Since the numbers are in ascending order, x is the number with the highest value here.

From the arrangement, we can see that the median (the middle number) is the third number which is 9

The mean is the sum of the numbers divided by their count. So we set up the mean and equate to the median

We have this as;

(1 + 9 + 9 + 10 + x)/5 = 9

29 + x = 5(9)

29 + x = 45

x = 45-29

x = 16

Find m20. The diagram is not to scale.
R
689
33°
Select one:
O a. 79
O b. 101
O c.
89
O d. 112

Answers

68+33=101
180-101= 79.
79 is the answer

Question is in the photo*

Answers

Answer:

3x - y +5=0

Step-by-step explanation:

Slope:-

=> m = 2-8/-1-1 = -6/-2

=> m = 3

Using point slope form :-

=> y - 2 = 3 ( x - (-1))

=> y - 2 = 3x + 3

=> 3x - y +5 = 0

Answer:

y=3x+5

Step-by-step explanation:

first find the gradient

m=y2-y1/x2-x1

from the coordinates you are given x one is -1,x two is1,y one is 2,y two is 8

8-2/1-(-1)

=3

therefore the gradient is 3

after finding the gradient use the equation

y-y1=m(x-x1)

substitute x1,y1 and the gradient..y and x are constant

y-2=3(x-(-1))

y-2=3x+3

y=3x+3+2

y=3x+5

10 points!!!!!:)

Describe the graph of the function.

[tex]y = \sqrt{x + 3} [/tex]
a). the graph is a radical function y=sqrt(x) shifted 3 units right.
b). the graph is a radical function y=sqrt(x) 3 units left.
c). the graph is of exponential growth.
d). the graph is an absolute value function with vertex (-3,0).

pls help me:).

Answers

Answer:

b) y=sqrt(x) shifted 3 units left

Step-by-step explanation:

3rd line in the attached table:

f(x+3) = sqrt(x+3) horizontal translation left 3 units

Please decide !!!!!!

Answers

A + B + C = 13

Starting from the top and going horizontally the missing numbers are:
18, 3, 1, 9, 7, 6, 2, 8, 16, 4, 15, 10

The sum of 1+2+3+...+18+19 = 190
There are 5 vertical lines (3,4,5,4,3 in each) so each line has to add to 190 / 5 = 38

The equation of a line is 2(y + 1) = 10x + 3.
The y-intercept of the line is
, and the slope of the line is

Answers

Answer:

y intercept is (0,-0.5) and the slope is 5.

Step-by-step explanation:

Let's rewrite the equation in slope intercept form.

Step 1: divide both sides by two:

[tex]\frac{2\left(y+1\right)}{2}=\frac{10x}{2}+\frac{3}{2}[/tex]

Step 2: simplify [tex]y+1=\frac{10x+3}{2}[/tex]

Step 3: subtract one from both sides: [tex]y+1-1=\frac{10x+3}{2}-1[/tex]

[tex]y=\frac{10x+3}{2}-1[/tex]

[tex]y = 5x - \frac{1}{2}[/tex]

Solve for X.

-3/2(x-2)=45/14

Please show work.

Answers

Answer:

-1/7

Step-by-step explanation:

-3/2(x-2)=45/14

x-2=(45/14)/(-3/2)

x-2=(45/14)(-2/3)

x-2=-90/42

simplify -90/42 into -15/7

x-2=-15/7

x=-15/7+2

x=-15/7+14/7

x=-1/7

Exact Form:
x=-1/7

Decimal form:

-0.142857

Explanation:
You simplify both sides of the equation and then isolate the variable.

5- Observá el cartel y resolvé calculando el m.C.M entre las cantidades. FRECUENCIA A Mar del Plata, cada 2 horas A Necochea , cada 4 horas A San Clemente, cada 6 horas a) Si acaba de partir un micro a cada destino, ¿cuántas horas pasarán para que vuelvan a salir tres micros al mismo tiempo?

Answers

Respuesta:

12 horas

Explicación paso a paso:

Dado que :

A Mar del Plata = cada 2 horas

A Necochea = cada 4 horas

A San Clemente = cada 6 horas

Número de horas que tardarán los 3 autobuses en salir al mismo tiempo, si ambos salieron ahora;

Tome el mínimo común múltiplo de los 3 autobuses:

Múltiplos de:

2: 2, 4, 6, 8, 10, 12, 14, 16, 18, 20, 22

4: 4, 8, 12, 16, 20, 24, 28, 32

6: 6, 12, 18, 24, 30, 36, 42

El mínimo común múltiplo de 2 horas, 4 horas y 6 horas es 12;

Por lo tanto, los 3 autobuses saldrán a la misma hora en las próximas 12 horas:

Which formulas can be used to find the surface area of a right prism where p is the perimeter of the base, h is the height of the prism, BA is the area of bases, and LA is the lateral area? Check all that apply.
A. SA = BA - LA
B. SA = p + LA
C. SA = BA + LA
D. SA = BA + ph
E. SA = 1 / BA + LA​

Answers

Answer:

SA=BA+LA and SA=BA+ph

Step-by-step explanation:

I just looked it up

The correct formulas to find the surface area of a right prism are:

SA = BA + LA and SA = BA + ph.

Options (A), and (D) are the correct answer.

What is a prism?

A prism is a three-dimensional object.

There are triangular prism and rectangular prism.

We have,

The correct formulas to find the surface area of a right prism are:

1)

SA = BA + LA, where SA is the total surface area, BA is the area of the two identical bases, and LA is the lateral area (the sum of the areas of all the rectangular sides).

2)

SA = BA + ph, where SA is the total surface area, B is the area of one base, p is the perimeter of the base, h is the height of the prism, and ph is the area of all the rectangular sides (the lateral area).

Therefore,

The correct formulas to find the surface area of a right prism are:

SA = BA + LA and SA = BA + ph.

Learn more about prism here:

https://brainly.com/question/12649592

#SPJ7

Graph the line with slope -5 and y-intercept 1.

Answers

Answer:y=-5+1

Step-by-step explanation:

Other Questions
What is a migrant worker? traveling agriculture worker a skilled factory workerlaborer with permission to work in a foreign nationO a worker in a maquiladora Exhibit 15-8 The following estimated regression model was developed relating yearly income (y in $1000s) of 30 individuals with their age (x1) and their gender (x2) (0 if male and 1 if female). = 30 + 0.7x1 + 3x2 Also provided are SST = 1200 and SSE = 384. The yearly income of a 24-year-old female individual is _____. a. $49.80 b. $19,800 c. $19.80 d. $49,800 The two triangles below are congruent. Which of the following are congruent by CPCTC? help me out please, thank you 50 POINTSSS!!! I NEED HELP ASAPPP!!!Choose one of these topic sentences and write a paragraph using at least three adjectives, three adverbs, and three prepositional phrases.1. If I were seven feet tall, I'd . . .2. I believe in miracles.3. Money cannot buy the best things in life. The table shows the altitudes of four different cities. What is the correct way to arrange the altitudes in decreasing order?City Altitude (feet)Snowvale 13Highbridge 58Westsilver -51Springmoor -35A. 58, 13, -35, -51B. -51, -35, 13, 58C. -35, -51, 13, 58D. 58, 13, -51, -35 Given the following angles, what ray is the common side of CFD and ZDFE? A grade 12 Physics student shoots a basketballfrom the ground at a hoop which is 2.0 m aboveher release. The shot was at a velocity of 10 m/sand at an angle of 80 to the ground.a. Determine the vertical velocity of the ballwhen it is at the level of the net. Youshould get two answers. Please show ALL steps Please help me asap! Two parallel conducting plates are separated by 12.0 cm, and one of them is taken to be at zero volts. (a) What is the magnitude of the electric field strength between them, if the potential 5.6 cm from the zero volt plate is 450 V Which of the following is the equation of the quadratic function below? In the diagram below, trapezoid ABCD maps to trapezoid ABCDWhich angle corresponds to angle C cuantas estrofas tiene el poema el vuelo de las garzas plz ayuda line F has a slope of. -6/3 and line G has a slope of -8/4. what can be determined about distinct lines F and G? Sobre que superficie se desplazara mas rpidamente un tejo, sobre cemento o sobre cermica? por que? Why do gases act more ideal atlower pressures? Area of this figure You and your friend are playing a game by tosing two coins land on different sides your friend wins the table shows the possible outcome? Is this a fair game help me if tou don't then no brainlist for you if x = 3 and y = 2, then the value of ( x - y )(x + xy + y) is(a) 31 (b) 19 (c) 1 (d) 25